Area Under Hyperbola: Green's Theorem

  • Thread starter Thread starter sara_87
  • Start date Start date
  • Tags Tags
    Area Hyperbola
Click For Summary
The discussion focuses on finding the area enclosed by the hyperbola 25x^2 - 4y^2 = 100 and the line x = 3 using Green's Theorem. Participants clarify the use of the area formula area = 1/2 ∫(x dy - y dx) and discuss the necessary parametrizations for both the hyperbola and the line. The importance of maintaining consistent orientation for the line integrals is emphasized, and it is suggested to perform two separate line integrals for clarity. The integrand changes significantly when parametrizing the linear boundary, prompting questions about how to correctly incorporate these into the calculations. Overall, the conversation revolves around the correct application of Green's Theorem to solve the problem effectively.
sara_87
Messages
748
Reaction score
0

Homework Statement



Find the area enclosed by the hyperbola: 25x^2-4y^2=100 and the line x=3
using the green's theorem

Homework Equations



Green's theorem:
\int_C[Pdx+Qdy]=\int\int(\frac{\partial Q}{\partial x}-\frac{\partial P}{\partial y})dxdy

The Attempt at a Solution



We can write the area of the domain as:
area=\frac{1}{2}\int(xdy-ydx)
I know what the graph looks like and i know the parametrisation:
x=2cosht
y=bsinht
but i am to use: area=\frac{1}{2}\int(xdy-ydx) what would be the limits of integration?
 
Physics news on Phys.org
The t limits for the hyperbolic segment of the parametrization are where x=3, i.e. 3=2*cosh(t), yes? Don't forget you need a separate parametrization for the linear part of the boundary x=3 and don't forget to choose a consistent orientation for the two line integrals.
 
thanks for the limits, i agree.
When i parametrize the linear part at the boundary x=3, how does this effect the integrand?
 
sara_87 said:
thanks for the limits, i agree.
When i parametrize the linear part at the boundary x=3, how does this effect the integrand?

The integrand is completely different. To do the line part you need to write an x(t) and y(t) that parametrize the line x=3.
 
Oh right i see. so when i do that, when i find x(t) and y(t) for the line, and the X(t) and Y(t) for the hyperbola part, how do i out this in the integrand?
I mean for the xdy part, is this: (x(t)+X(t))dy(t)
?
 
sara_87 said:
Oh right i see. so when i do that, when i find x(t) and y(t) for the line, and the X(t) and Y(t) for the hyperbola part, how do i out this in the integrand?
I mean for the xdy part, is this: (x(t)+X(t))dy(t)
?

Why don't you just do two separate line integrals instead of trying to mix them up? That's what I would do.
 
Question: A clock's minute hand has length 4 and its hour hand has length 3. What is the distance between the tips at the moment when it is increasing most rapidly?(Putnam Exam Question) Answer: Making assumption that both the hands moves at constant angular velocities, the answer is ## \sqrt{7} .## But don't you think this assumption is somewhat doubtful and wrong?

Similar threads

Replies
3
Views
2K
  • · Replies 2 ·
Replies
2
Views
2K
  • · Replies 7 ·
Replies
7
Views
2K
  • · Replies 2 ·
Replies
2
Views
1K
  • · Replies 1 ·
Replies
1
Views
2K
Replies
4
Views
3K
  • · Replies 2 ·
Replies
2
Views
1K
  • · Replies 6 ·
Replies
6
Views
3K
  • · Replies 4 ·
Replies
4
Views
3K
  • · Replies 8 ·
Replies
8
Views
2K